JEE Main & Advanced Mathematics Binomial Theorem and Mathematical Induction Question Bank Multinomial theorem, Terms free from radical sign in the expansion(a1/p+b1/q), Problems regarding to three four consecutive terms or coefficients

  • question_answer
    The greatest integer less than or equal to \[{{(\sqrt{2}+1)}^{6}}\]is [RPET 2000]

    A) 196

    B) 197

    C) 198

    D) 199

    Correct Answer: B

    Solution :

    Let \[{{(\sqrt{2}+1)}^{6}}=k+f,\]where k is integral part and f the fraction\[(0\le f<1)\]. Let\[{{(\sqrt{2}-1)}^{6}}=f',\,(0<f'<1)\], Since \[0<(\sqrt{2}-1)<1\] Now, \[k+f+{f}'={{(\sqrt{2}+1)}^{6}}+{{(\sqrt{2}-1)}^{6}}\] \[=2\left\{ {{\,}^{6}}{{C}_{0}}{{.2}^{3}}+{{\,}^{6}}{{C}_{2}}{{.2}^{2}}+{{\,}^{6}}{{C}_{4}}.2+{{\,}^{6}}{{C}_{6}} \right\}=198\] ?..(i) \[\therefore \,\,\,f+{f}'=198-k=\] an integer But, \[0\le f<1\]and \[0<{f}'<1\Rightarrow 0<(f+{f}')<2\] \[\Rightarrow f+{f}'=1\],      \[(\because f+{f}'\]is an integer) \ By (i), I =198 ? (\[f+{f}'\]) = 198  ? 1 = 197.


You need to login to perform this action.
You will be redirected in 3 sec spinner